0
$\begingroup$

I'm trying to obtain a parallel shift in my efficient frontier based on the Merton 1972-parameters. As i think a picture tells you more than 1000 words here is what i tried:

The setting of my problem is the following:

Correlation Matrix:

enter image description here

Std.and Mean vector:

enter image description here

To derive an analytical solution for the efficient frontier (as i don't want to numerically search every time i try something) i obtained the following frontier:

enter image description here

I try to shift this efficient frontier to the right (e.g. increasing the volatility of all combinations of assets). Here is an illustration. The two green frontiers are what i get if i use the original market setting and if i shift the volatility of the efficient frontier by a fixed shift (here by 5% to illustrate my problem).

The blue line is the analytical frontier that i hope to get after changing the covariance matrix (after recalculating my Merton 1972-Parameters given the changed covariance matrix) such that it matches with the shifted efficient frontier (the right one in green).

enter image description here

I tried to shift the volatility-vector but this changed the curvature and the position of the blue efficient frontier but doesn't match the right green one.

My question is therfore: How can i manipulate the covariance matrix such that my blue efficient frontier matches the right green frontier?

Please let me know if something is not clear, i will provide details then. As always i appreciate your help and suggestions. Thomas

EDIT: I played around a bit and here is one thought: Minimizing the sum of squared differences by manipulating my Merton parameters gets me somehow close.

enter image description here

Any better ideas? :-/

$\endgroup$
2
  • $\begingroup$ Can you change means instead of variances and covariances? $\endgroup$ Commented Sep 20, 2023 at 13:59
  • $\begingroup$ @phdstudent Unfortunately not, but i can manipulate the Merton parameters if i can't find a solution for the covariance matrix for the moment, but the ultimate target is a modification of the covariance matrix. In addition, a shift in the mean vector would shift the efficient frontier up or down, correct? $\endgroup$ Commented Sep 20, 2023 at 14:06

0

Your Answer

By clicking “Post Your Answer”, you agree to our terms of service and acknowledge you have read our privacy policy.

Start asking to get answers

Find the answer to your question by asking.

Ask question

Explore related questions

See similar questions with these tags.